Difference between revisions of "2012 AMC 8 Problems/Problem 15"

Line 1: Line 1:
 
The smallest number greater than 2 that leaves a remainder of 2 when divided by 3, 4, 5, or 6 lies between what numbers?
 
The smallest number greater than 2 that leaves a remainder of 2 when divided by 3, 4, 5, or 6 lies between what numbers?
  
<math> \textbf{(A)}\hspace{.05in}40\text{ and }50\qquad\textbf{(B)}\hspace{.05in}51\text{ and }55\qquad\textbf{(C)}\hspace{.05in}56\text{ and }60\qquad\textbf{(D)}\hspace{.05in}\text{61 and 65}\qquad\textbf{(E)}\hspace{.05in}\text{66 and 99} </math>
+
<math> \textbf{(A)}\hspace{.05in}40\text{ and }50\qquad\textbf{(B)}\hspace{.05in}51\text{ and }55\qquad\textbf{(C)}\hspace{.05in}56\text{ and }60\qquad\textbf{(D)}\hspace{.05in}61\text{ and }65\qquad\textbf{(E)}\hspace{.05in}66\text{ and }99 </math>
 +
 
 +
==Solution==
 +
To find the answer to this problem, we need to find the least common multiple of <math>3</math>, <math>4</math>, <math>5</math>, <math>6</math> and add <math>2</math> to the result. The least common multiple of the four number is <math>60</math>, and adding <math>2</math>, we find the number we want is <math>62</math>. Now we need to find the range which contains <math>62</math>. The only such range and our final answer is <math> \boxed{\textbf{(D)}\ 61\text{ and }65} </math>.
  
 
==See Also==
 
==See Also==
 
{{AMC8 box|year=2012|num-b=14|num-a=16}}
 
{{AMC8 box|year=2012|num-b=14|num-a=16}}

Revision as of 12:09, 24 November 2012

The smallest number greater than 2 that leaves a remainder of 2 when divided by 3, 4, 5, or 6 lies between what numbers?

$\textbf{(A)}\hspace{.05in}40\text{ and }50\qquad\textbf{(B)}\hspace{.05in}51\text{ and }55\qquad\textbf{(C)}\hspace{.05in}56\text{ and }60\qquad\textbf{(D)}\hspace{.05in}61\text{ and }65\qquad\textbf{(E)}\hspace{.05in}66\text{ and }99$

Solution

To find the answer to this problem, we need to find the least common multiple of $3$, $4$, $5$, $6$ and add $2$ to the result. The least common multiple of the four number is $60$, and adding $2$, we find the number we want is $62$. Now we need to find the range which contains $62$. The only such range and our final answer is $\boxed{\textbf{(D)}\ 61\text{ and }65}$.

See Also

2012 AMC 8 (ProblemsAnswer KeyResources)
Preceded by
Problem 14
Followed by
Problem 16
1 2 3 4 5 6 7 8 9 10 11 12 13 14 15 16 17 18 19 20 21 22 23 24 25
All AJHSME/AMC 8 Problems and Solutions